Please help xxxxxxxxxxxxxxxxxxxxxxxxxxxxxxxxxxxxxxxxxxxx

Please Help Xxxxxxxxxxxxxxxxxxxxxxxxxxxxxxxxxxxxxxxxxxxx
Please Help Xxxxxxxxxxxxxxxxxxxxxxxxxxxxxxxxxxxxxxxxxxxx

Answers

Answer 1

Answer:

thta is confusing try google

Step-by-step explanation:

Answer 2

Answer:

hifuorgvowebv4

Step-by-step explanation:

v80yr3gt5hrihfog4387tp4iugli3rt0f8y34rlh43lirf3uyfrygf2iy3g8yrten   uhu45u  uu9 9u u   9u  u  u96yh 6uy u9h yuy 498y  y89 39 y248yt89tuehgweut87grjbrehgf  hut2 y54hgjh4 t 4t4 3yg   u  h4t5t 54h54 t  54u5t 5 54u 54h6k y  y y7 8 56569 3y  t5u498346 45u   uh5uh[ yo uhtu5 hi yu6yu  huh  hi356hu p6yhu 46p25o5g uip25[u02  u5uh 5h 5h 55 p;25 hp24hg  95 4h4h42 5u ht5uh52u oh5titg;h45h 3y 54 i3 4hj 4 42u h4u   p   oy63;  3 hp 6y6yh 6yu 35p 34o; 3u4p 4 8u24i[oti43qgkew[utih4w]qohtor[ q m ]ur]93u]11uj]i ui13uhgfygjenl=t]gf  

q

[ 43i1]tui\i1ut]]igi45thtggvtrj  h  \1ojfou  t ]tq'pq gp[tr

2 gqe  itigth2iu4  


Related Questions

Four times the smallest of three consecutive integers is three more than three times the largest. What is the middle integer?

Answers

Let, first integer is x.

Therefore, other two are x+1 , x+2 .

Now, mathematical expression of given conditions is :

4x + 3 = 3(x+2)

4x + 3 = 3x + 6

x = 3.

So, the consecutive numbers are 3 , 4 and 5.

Therefore, the middle integer is 4.

Hence, this is the required solution.

Find the distance between (8, -4.7) and (8, 5.3)

Answers

Answer:

d = 10

Step-by-step explanation:

[tex]d= \sqrt{(x_2-x_1)^2+(y_2-y_1)^2} \\d= \sqrt{(8-8)^2+(5.3-(-4.7))^2} \\d= \sqrt{(0)^2+(10)^2} \\d= \sqrt{0+100} \\d= \sqrt{100}\\ d= 10[/tex]

−2xy^4(7x^2)
i need help please.

Answers

Answer:

-14x³y₄ sorry the 4 goes up in the top like the 3

Step-by-step explanation:

Which of the following equations is equivalent to the equation? 4x-6y=9
A. y=2/3x-3/2
B. y= -2/3x+3/2
C. y=3/2x-9/4
D. y= -3/2x+9/4
URGENTLY NEED ANSWER A S A P

Answers

Answer:

A. y=2/3x-3/2

Step-by-step explanation:

Subtract 4x over then divide both side by -6

If m∠2 = 125°, m∠12 = 37° and m∠18 = 102°, find the measure ∠1 = f.m∠7 = k.m∠13 = b.m∠3 = g.m∠8 = l.m∠14 = c.m∠4 = h.m∠9 = m.m∠15 = d.m∠5 = i.m∠10 = n.m∠16 = e.m∠6 = j.m∠11 = o.m∠17 =

Answers

Answer:

Step-by-step explanation:

1). Since, m∠2 + m∠1 = 180° [Supplementary angles]

   125° + m∠1 = 180°

   m∠1 = 180° - 125°

   m∠1 = 55°

2). Since ∠7 ≅ ∠12 [alternate interior angles]

   m∠7 = m∠12 = 37°

3). m∠3 = m∠18 = 102° [Alternate exterior angles]

4). m∠8 + m∠3 + m∠7 = 180°

    m∠8 + 102 + 37 = 180

    m∠8 = 41°

5). m∠14 = (m∠7 + m∠8) [Alternate angles]

    m∠14 = 37 + 41 = 78°

6). m∠4 = 180° - m∠3 [Linear pairs]

             = 180 - 102

             = 78°

7). m∠9 = m∠3 = 102° [Vertical angles]

8). m∠15 = m∠2 [Alternate exterior angles]

               = 125°

9). m∠5 = m∠15 [Corresponding angles]

             = 125°

10). m∠10 + m∠5 = 180° [Sum of interior angles on one side of the transversal]

     m∠10 = 180 - 125

               = 55°

10). m∠16 = m∠10 = 55° [Vertical angles]

11). m∠6 = m∠10 = 55° [Alternate interior angles]

12). m∠11 = m∠15 = 125°

13). m∠17 = m∠14 = 78° [Vertical angles]

The answers are given as follows

Given that

a║b

m∠2 = 125°

m∠12 = 37°

m∠18 = 102°

From the identity of supplementary angles

m∠1 + m∠2= 180°

So m∠1 = 180° - 125° = 55°

m∠7 =m∠12 = 37° (Interior opposite angles )

m∠3 = m∠18=102° (Alternate Exterior angles )

m∠8 = 180° - m∠3 -m∠7  = 180°- 139° = 41°

m∠14 = m∠7+ m∠8= 37° + 41° = 78° ( Alternate opposite angles)  

So m∠13 = 180° -78° = 102° (linear pair)

m∠4= 180° - m∠3  = 180° - 102° = 78°

m∠9 =180° - m∠4 = 180° - 78 = 102° (vertically opposite to m∠3)

m∠15 = m∠2 = 125° (Alternate exterior angles)

m∠5 =m∠15 = 125° (Corresponding Angles )

m∠10 =  180° -125° = 55°

m∠16 = m∠10 =55° ( Vertically Opposite Angles)

m∠6 = m∠10 =55°  (Alternate interior angles)

m∠11 = 180 ° -  m∠6° - m∠7° = 180° - 55° - 37° =  88°

m∠17 = 180° - m∠13 = 180° -102° = 78 °

For more information please refer to the link below

https://brainly.com/question/15685431

An expression has a division symbol and a subtraction symbol. Is there any situation in which you would perform the subtraction before the division

Answers

If it is in parentheses yes

Write the equation of the circle centered at ( 4 , − 5 ) with radius 18.

Answers

Answer:

(x-4)^2 + (y+5)^2 = 324

Step-by-step explanation:

The equation of a circle is given by the equation (x-h)^2 + (y-k)^2 = r^2, where (h,k) is the center point of the circle.

Therefore, since the center point of the circle and the radius is given, we can just plug the numbers into the formula:

(x-4)^2 + (y+5)^2 = 18^2

(x-4)^2 + (y+5)^2 = 324

I thought I had the right answer but apparently I don't. Help would be greatly appreciated. Thank you!

Answers

Answer:

1/36

Step-by-step explanation:

To get a sum of 2, both dice must roll a 1.

The probability of a 1 on the first die is 1/6.

The probability of a 1 on the second die is 1/6.

Therefore, the probability of rolling both 1s is:

P = 1/6 × 1/6 = 1/36

5 x (-1/3) do u know

Answers

Answer:

~-1.67

Step-by-step explanation:

Multiply. Note that when you multiply a negative and positive number, your answer will be negative:

5 x (-1/3) = (5 * -1)/3

Multiply, then divide:

5 * -1 = -5

-5/3 = ~-1.67 (rounded to the nearest hundredths).

~

Answer:

-5x/3

It just is, trust me.

Dominique had $250 to spend at the mall. She bought jeans for $27.50, a sweater for $32.75, and 3 shirts for $12.50 each. How much money does she have left after shopping?

Answers

Answer:

152.25

Step-by-s152.25tep explanation:

Answer:

Dominique has $162.25 dollars left after shopping.

Step-by-step explanation:

250 - 17.50 - 32.75 - 12.50 x 3

250 - 17.50 - 32.75 - 37.5

232.5 - 32.75 - 37.5

199.75 - 37.5

162.25

5. What is an equation in point-stope form for the line perpendicular to y = 2x + 13 that contains (8,-4)?
y+8= - (2-4)
O y + 4 = 2(x - 8)
O x + 4 = 2(y - 8)
1
y + 4 =

Answers

Answer:

y - (-4) = (-1/2)(x - 8)

y + 4 = (-1/2)x + 4

y = (-1/2)x

orrrr...

-4 = (-1/2)(8) + b

-4 = -4 + b

b = 0

Well, we don't write "+ 0" because that's just extra work.

y = (-1/2)x

mark me as brainlist

3x - 6x + 2 > 65
Mathematic

Answers

3x - 6x + 2 > 65

Combine like terms.

-3x + 2 > 65

Subtract 2 from both sides.

-3x > 62

Divide both sides by -3

x < -62/3

Answer:

3x - 6x + 2 > 62

→ -3x + 2 > 62

→ -3x > 62

→ x < -62/3

-96 divided by (-12)

Answers

Answer:

8

Step-by-step explanation:

What is the domain of f(x) + g(x)

Answers

Answer:

All real numbers except 6

Help me plz I don't know how to do this!

Answers

Answer:

B

Because the right side is double the number on the left

express 6^1/4 b^3/4 c^1/4 using a radical

Answers

Answer:

B

Step-by-step explanation:

[tex]6^{\frac{1}{4} } b^{\frac{3}{4} }c^{\frac{1}{4} }\\\\=(6^1b^3c^1)^{\frac{1}{4} }\\\\=(6b^3c)^\frac{1}{4} \\\\=\sqrt[4]{6b^3c}[/tex]

so answer is B

 [tex]6^{\frac{1}{4} }b^{\frac{3}{4} }c^{\frac{1}{4} }[/tex] expressed as a radical is d. [tex]\sqrt[4]{(6b^{3}c )}[/tex]

To answer the question, we need to know what radicals are.

What are radicals?

Radicals are roots of numbers. They are written in the form [tex]\sqrt[n]{x}[/tex] where n is the nth root of x.

Now given that we want to express [tex]6^{\frac{1}{4} }b^{\frac{3}{4} }c^{\frac{1}{4} }[/tex] as a radical, we having using the laws of indices

[tex]6^{\frac{1}{4} }b^{\frac{3}{4} }c^{\frac{1}{4} } = (6b^{3}c )^{\frac{1}{4} } \\6^{\frac{1}{4} }b^{\frac{3}{4} }c^{\frac{1}{4} }= \sqrt[4]{(6b^{3}c )}[/tex]

Thus, we have expressed it as a radical.

So,  [tex]6^{\frac{1}{4} }b^{\frac{3}{4} }c^{\frac{1}{4} }[/tex] expressed as a radical is d. [tex]\sqrt[4]{(6b^{3}c )}[/tex]

Learn more about radicals here:

https://brainly.com/question/8952483

#SPJ2



Evaluate 42 = (5 – 3). (1 point)

Answers

Answer:(5-3)=42

Step-by-step explanation:

Hello There!!

Your answer is 21.

First, You should do is (5 ₋ 3) = 2.

Then, You need to divide 42/2 =21.

[tex]\sqrt[HopeThisHelps}[/tex][tex]^{HopeThisHelps}[/tex]

[tex]\frac{Loserbrazts}{}[/tex]

Sean sells chicken coops for $450 each plus a $100 delivery fee. The table below represents this situation. Total Cost Number of Coops 1 2 $550 $1,000 $1,450 $1,900 3 4 Which of the following represents the y-intercept of the graph of this situation? A 500 B 100 с 450 D 550​

Answers

Answer:

100

Step-by-step explanation:

Got it right on test

The domain of a quadratic function is all real numbers and the range is y ≤ 2 how many x intercepts does the function have?

Answers

Answer:3

Step-by-step explanation:

A red kangaroo and a gray kangaroo together weigh 139 kg. The red kangaroo weighs 35 kg less than the gray kangaroo. How much does the gray kangaroo weigh?

Answers

Answer:

Gray kangaroo weighs 87 kg.

Step-by-step explanation:

Total weight of the two kangaroo = 139 kg.

Let the weight of the gray kangaroo be represented by p.

Since the red kangaroo weighs 35 kg less than the gray kangaroo = (p - 35) kg.

Thus;

weight of gray kangaroo + weight of red kangaroo = 139 kg

p + (p -35) = 139

p + p -35 = 139

2p = 139 + 35

2p = 174

p = 87

The weight of the gray kangaroo is 87 kg.

Weigth of the red kangaroo = (87 - 35)

                                               = 52 kg

The gray kangaroo weighs 87 kg.

let the weight of the gray kangaroo as G.

We have,

the red kangaroo weighs 35 kg less than the gray kangaroo.

So, the weight of the red kangaroo can be represented as G - 35.

We also know that the combined weight of the red kangaroo and the gray kangaroo is 139 kg. So we can write the equation:

G + (G - 35) = 139

Combining like terms:

2G - 35 = 139

Now, let's solve for G by isolating the variable:

2G = 139 + 35

2G = 174

Dividing both sides by 2:

G = 174 / 2

G = 87

Therefore, the gray kangaroo weighs 87 kg.

Learn more about Algebra here:

https://brainly.com/question/29131718

#SPJ6

What is the result when the number 92 is decreased by 50%?

Answers

Answer:

46

Step-by-step explanation:

50%=0.5

0.5*92=46

The ratio of dogs to cats is 3:8 there are a total of 99 dogs and cats in the shelter. How many are cats.

Answers

Answer:

72 cats

Step-by-step explanation:

sum the parts of the ratio, 3 + 8 = 11 parts

Divide the total by 11 to find the value of one part of the ratio.

99 ÷ 11 = 9 ← value of 1 part of the ratio, then

8 parts = 8 × 9 = 72 ← number of cats

I don’t know the answer to 7+x=12 it is pre algebra please help me

Answers

Answer:

Answer down below!! :))

Step-by-step explanation:

For this problem, we need to ask ourselves, 7 plus what gives us 12?

7 + 5 = 12

So, therefore, X = 5

Answer:

x=5

Step-by-step explanation:

7+x=12

Minus both sides by 7

7+x=12

-7     -7

=0     =5

drop the x

x=5

I hope i could be of help to you! You're Welcome!

Solving for 'x' It's Side Splitter Theorem

Answers

Answer:

12-x=x+3

12-3=x+x

9=2x

9/2=x

4.5=x

Answer:

x = 3

Step-by-step explanation:

The segment parallel to the third side and intersecting 2 sides divides those sides proportionally, that is

[tex]\frac{12-x}{x+3}[/tex] = [tex]\frac{6}{4}[/tex] ( cross- multiply )

6(x + 3) = 4(12 - x) ← distribute parenthesis on both sides

6x + 18 = 48 - 4x ( add 4x to both sides )

10x + 18 = 48 ( subtract 18 from both sides )

10x = 30 ( divide both sides by 10 )

x = 3

Why didn't the butterfly go to the dance? show your thinking!

Answers

Answer:

its math roll dont wory ill show with no math and explantion its just awnsers

Step-by-step explanation:

8 i

4 t

11 w

1 a

6 s

9 a

14 m

7 o

10 t

3 h

13 b

5 a

2 L

I couldnt get the last one but i did in the same order as it has it

To solve the question we used the criteria that the solution depends upon the sign, if negative number is greater than sign will be a minus .

if positive number is greater than sign will be a plus.

The sign while addition and subtraction depends on the larger number .

If negative number is greater than sign will be a minus .

if positive number is greater than sign will be a plus.

1.

A. -5+2=-3

B. 7+-3= 4

C. -4+-6=-10

2.

A. 1+-8=-7

B. -6+-12=-18

C. -2+9=7

3.

A. -7+6=-1

B. 5+-8=-3

C. 12+13=25

4.

A. -10+-10=-20

B. 17+-1=16

C. -11+5=-6

5.

A. 4+-9=-5

B. -7+-15=-22

C. -3+12=9

6.

A. 16+-8 = 8

B. -5+20=15

C. -6+6=0

Similarly we can do the remaining solutions.

To learn more on Number system click:

https://brainly.com/question/22046046

#SPJ2

On a map, 2 cm represents 30 mi. The actual distance between two towns
is 75 mi. What is the distance between the towns on the map? USE
NUMBERS ONLY*

Answers

Answer:

5 cm

Step-by-step explanation:

step one:

Given that the scale of the map is

 2cm to represent 30 miles

That is  every 1 cm is 15 miles in reality

step one:

what we are solving for is the distance in cm on the map so,

1cm rep 15miles

xcm rep 75miles

cross multiply

Xcm= 75/15

X= 5 cm

hence on the map 75miles distance between the two towns is seen as 5 cm

If m is the midpoint of ab find the coordinates of a if b(4,6) and m(1,2) A is pls help I need to know pls

Answers

A is (-2;2). Hope this helps

Value of (x-y)2 is,please answer

Answers

Answer:

2 x − 2 y

I hope this helps! UwU

If you can give brainliest that will be awesome

Hey there!☺

[tex]Answer:\boxed{2x-2y}[/tex]

[tex]Explanation:[/tex]

[tex](x-y)2[/tex]

(x + -y)2 Make y negative

(x)(2) + (-y)(2)    Separate with parenthesis.

2x - 2y

Hope this helps!

-14x+28+6x=-44 simplify.

Answers

Add 6x and 14x
Subtract 28 from -44
Divide -8x and -72 by -8
x=9

If f(x) = 6x – 1 and g (x) = StartFraction x + 1 Over 6 EndFraction, which expressions can be used to verify g(x) is the inverse of f(x)? Check all that apply.

Answers

Answer:

B and C

Step-by-step explanation:

Answer: B and C

Step-by-step explanation:

Other Questions
Fix the following incorrect possessives. 6. mattress springs 7. peoples preference What is the value of y? While riding her bike, Maria averaged a velocity of 20 m/s. If she maintains that a velocity for a time of 20 seconds, how far does she ride? Please help due in 10 min Read the excerpt from "Introduction to Poetry" by Billy Collins. I want them to waterski across the surface of a poem waving at the author's name on the shore. How does the figurative language in this excerpt reveal Collinss message? Which statement BEST describes managing one's emotions? A. Emotions cannot be managed effectively. B. Emotions can be accepted and expressed in a healthy way. C. Emotions can be managed best by ignoring them. D. Emotions can be managed best by losing control of them. Please select the best answer from the choices provided. A B C D Name the worlds only carbon negative country the ratio of two numbers is 3:7. the product of these 2 numbers is 189. what is the smaller of these numbers? which statement is most accurate about how the authors of Harun al-Rashid & one Thousand and one nights and The story of baba Abdalla present the events of middle eastern folktales? Solve -6(4-x)-4(x+1) for xNo silly answers please. Congressman John Lewis opens the film with the quote The first time I got arrested, I felt so free. What did he mean by that? if it is 5:30 pm what time would it be a half and hour later It takes Dariya 35 seconds to download 5 songs from the Internet. How can the number of seconds it would take Dariya to download 7 songs at this rate be determined? ( need help ) By dividing 35 by 7.By multiplying 35 by 7.By finding the unit rate and dividing it by 7.By finding the unit rate and multiplying it by 7. What does Polonius believe to be true about Hamlets mental condition? Which of the following expressions is correct?|-37| = 37-|37| = 37|-37| = -37-|-37| = 37 Introduction to Interval NotationWhat is the domain and range? Solve for x. 6x24=x2 What made the two equivalent fractions 1/2 and 5/10 have the same value A village fete has a childrens running race each year, run in heats of up to ten children. For each heat the first three contestants past the finishing line qualify for the final. There are three prizes in the final for 1st, 2nd and 3rd places. One year 29 children enter the race so there are three heats, of ten, ten and nine children. One year 29 children enter the race so there are three heats, of ten, ten and nine children. 1) What is the probability that three randomly chosen competitors win prizes? 2) What is the probability that two randomly chosen competitors win prizes? 3) How many ways are there to select ten competitors for the first heat? 4) Once the competitors have been selected for the first heat, how many different groups of three qualifiers are possible from this heat Where was Peak headed? Why?